LSAT and Law School Admissions Forum

Get expert LSAT preparation and law school admissions advice from PowerScore Test Preparation.

 Administrator
PowerScore Staff
  • PowerScore Staff
  • Posts: 8950
  • Joined: Feb 02, 2011
|
#26207
Complete Question Explanation

Weaken, CE. The correct answer choice is (D)

The advertisement presented in this stimulus, presumably for the Lakeside Injury Clinic, concludes
that people should “insist on receiving Lakeside Injury Clinic’s complete course of treatment for
whiplash after any accident that involves a fall or bump on the head.” In support of this advice, the
advertisement points out that while whiplash injuries most commonly result from auto accidents,
whiplash can result from many different kinds of accidents that produce a sudden sharp motion of
the neck. As examples of events that can cause whiplash, the advertisement offers a fall, a bump on
the head, and being shoved from behind.

This is a causal argument. The advertisement essentially claims that because an accident that
involves a fall or a bump to the head can cause whiplash, then a person involved in such an accident
should be treated for whiplash, implying that they actually have whiplash. In other words, the
argument assumes that if something can cause an effect, then it does cause that effect.

The question stem identifies this as a Weaken question. Our prephrase is that the correct answer
choice will attack the flawed causal position that because a fall or a bump to the head can produce
whiplash, then it does induce whiplash and so any person who experiences any fall or a bump to the
head should receive Lakeside’s complete course of treatment.

Answer choice (A): While the stimulus mentioned being bumped from behind as an example of an
event that can cause whiplash, the conclusion was limited to only “a fall or a bump on the head.” So,
this answer choice is irrelevant to the conclusion.

Answer choice (B): This answer choice has no effect on the conclusion, because it does not tell us
whether auto accidents involving falling or being bumped on the head result in whiplash.

Answer choice (C): As with answer choice (B), this information is irrelevant to the conclusion
because it addresses accidents “other than those involving falls or bumps on the head,” which are
irrelevant to the conclusion.

Answer choice (D): This is the correct answer choice, because it directly weakens the author’s
assumption that since a fall or bump to the head can cause whiplash, then such an incident does
cause whiplash. If it is very uncommon for this type of event to result in whiplash, then it is not the
case that a person suffering from any fall or bump to the head should get Lakeside’s complete course
of treatment for whiplash.

Answer choice (E): This answer choice does not attack the conclusion because it discusses the
relevant treatment when whiplash actually occurs.
 JD180
  • Posts: 34
  • Joined: Aug 09, 2018
|
#50531
Hello,

For the correct answer (D), the explanation give above involves successfully weakening the argument. But the question asks 'which provides the strongest basis for criticizing the reasoning'. As I approached the question, I was looking for an answer that, would not allow the conclusion to be drawn (as I would with any question that has to do with asking how an argument is vulnerable to criticism).

Given that, I could not find an answer that did not allow the conclusion to be drawn, but instead I found and selected an answer (D) that weakened the reasoning.

My question to Powerscore then becomes about question type:

"Which of the following, if true, provides the strongest basis for criticizing the reasoning?"
Is this a WEAKEN question, or a 'vulnerable to criticism' question. My approach to each is different, and I can only surmise that its a weaken question even though it uses the word criticism.

How is my thinking?
 Sky Brooks
PowerScore Staff
  • PowerScore Staff
  • Posts: 18
  • Joined: Jul 14, 2018
|
#56791
Hi JD180,

For the "strongest criticism of the argument" question stem, you are looking for an answer choice which actively criticizes (or attacks) your argument and doesn't just point out a flaw. The answer choices in this question all introduce new specific details which could impact the argument, so you must identify which choice directly attacks the argument. In these types of questions, treat the problem like a weaken problem.

To "find the strongest basis for criticizing an argument", it is not necessary to disprove the argument. Instead you must identify the statement that if true, shows where the argument is weak.

Hope this helps!

-Sky Brooks
 JD180
  • Posts: 34
  • Joined: Aug 09, 2018
|
#56793
Sky Brooks wrote:Hi JD180,

For the "strongest criticism of the argument" question stem, you are looking for an answer choice which actively criticizes (or attacks) your argument and doesn't just point out a flaw. The answer choices in this question all introduce new specific details which could impact the argument, so you must identify which choice directly attacks the argument. In these types of questions, treat the problem like a weaken problem.

To "find the strongest basis for criticizing an argument", it is not necessary to disprove the argument. Instead you must identify the statement that if true, shows where the argument is weak.

Hope this helps!

-Sky Brooks
Thank you! This was very helpful.
 nihals23
  • Posts: 16
  • Joined: Oct 01, 2018
|
#59187
Hi Powerscore! I did not comprehend how D proved to be the correct answer choice. Could you please give a detailed explanation for the same? Thanks in advance.
 Robert Carroll
PowerScore Staff
  • PowerScore Staff
  • Posts: 1819
  • Joined: Dec 06, 2013
|
#60960
nihals,

Answer choice (D) is attacking the frequency of the situation in the third sentence. Even if falls and bumps on the head can produce the motion that might lead to whiplash, that doesn't prove it's so common that we should treat each such incident as if it has a substantial chance of having caused whiplash. But the conclusion claims one should get treated for whiplash in all these situations. So the conclusion is treating whiplash as likely enough in this situations to make treatment for whiplash appropriate. Answer choice (D) clarifies the third sentence and claims that whiplash is rare in those situations. Thus, it makes little sense to automatically treat for whiplash.

Robert Carroll
 lsatfighter
  • Posts: 26
  • Joined: Sep 26, 2018
|
#65689
I'm confused about E.

On one hand, I see that E strengthens the argument by giving us another reason to take Lakeside Injury Clinic's course.

On the other hand, it seems that E weakens the argument. If the treatment for whiplash caused by falls or bumps to the head is no different than the treatment for whiplash caused by auto accidents, then maybe there are other places or courses where you can go to receive the knowledge. For example, maybe an auto accident course from ANY clinic would suffice.

Is E wrong because it can either strengthen or weaken the argument, depending on how you look at it? Can someone please give me a detailed explanation of why E is wrong?
 George George
PowerScore Staff
  • PowerScore Staff
  • Posts: 48
  • Joined: Jun 07, 2019
|
#65865
@lsatfighter

The major problem in this argument's conclusion isn't the fact that it tells you which clinic to go to. The major problem in the conclusion is that it says we must "insist" on a "complete course of treatment for whiplash" after a fall or bump. However, the only premise (related to falls or bumps) says that falls or bumps "can" cause the sudden neck motion which result in whiplash. So the real problem (gap) in this argument is from the fact that bumps "can" cause injury/whiplash to the conclusion that we "must" get treatment "after any" bump or fall accident. The takeaway from this problem isn't that Lakeside Injury Clinic is trying to get a monopoly over competitors, but rather that they're trying to dupe people into seeking treatment when they may not necessarily need it. The takeaway that applies to other problems, @lsatfighter, is that you want to spot shifts in the verb tenses - from possibility to certainty. Changing the logical force from the premise (weak) to the conclusion (strong) is the real issue in this argument.

(E) is incorrect because the argument isn't concerned about comparing the kinds of treatment "appropriate" for different kinds of injuries. The argument is only concerned with establishing when someone should seek whiplash treatment, and recommending people who experience bumps and falls get that whiplash treatment. So, (E) is irrelevant. It has no impact on the argument. Further, you might label answer (E) as an irrelevant comparison. This kind of wrong answer trap compares two things along a dimension/factor the author doesn't care about. Here, whether the kinds of treatment "appropriate" for auto accidents is the same as the treatment for falls and bumps just doesn't matter.

While I didn't really agree with your line of reasoning above, @lsatfighter, (e.g. that (E) gives you additional reasons to get the clinic - It doesn't! - Just because the treatments would be the same, doesn't mean you should go and get them!), I do agree with your observation that an answer would be wrong if "it can either strengthen or weaken the argument, depending on how you look at it." Answers that can be manipulated for either side are incorrect on Weaken (and Strengthen) Qs on the LSAT.

Get the most out of your LSAT Prep Plus subscription.

Analyze and track your performance with our Testing and Analytics Package.